0 Daumen
5,5k Aufrufe


um zu zeigen, dass $$\lim_{n \rightarrow \infty} \frac{ln(n)}{n} = 0, ~n \in \mathbb{N}$$, reicht es da zu zeigen, dass der ln(n) immer langsamer wächst als n?

Das kann man zeigen mit

$$ln(n+1)-ln(n) < 1 \Leftrightarrow e^{ln(n+1) - ln(n)} < e \Leftrightarrow e^{ln(n+1)} \cdot e^{-ln(n)} < e \Leftrightarrow \frac{n+1}{n} < e \Leftrightarrow n+1 < e \cdot n \Leftrightarrow n > \frac{1}{e-1} \approx 0,6$$

Danke,

Thilo
Avatar von 4,3 k

"f wächst langsamer als g" ist die umgangssprachliche Version der Aussage lim f/g=0;

Die Folge an=n/2 erfüllt auch deine Ungleichung (sogar für alle n).

Dennoch ist  lim an/n=1/2 nicht 0.

Also funktioniert das so nicht.

 

Es gibt einige Varianten wie man das beweisen kann, z.B. über L'hopital oder mittels lim n1/n=1

Die Folge an=n/2 erfüllt auch deine Ungleichung

Welche Ungleichung erfüllt an=n/2 ?

Bitte mal kurz erläutern, ich verstehe nicht ganz, was gemeint ist ...

Die Ungleichung hier: ln(n+1)−ln(n)<1

So wie ich die Argumentation verstanden habe war wohl folgendes gedacht:

Aus an+1-an<1 für fast alle n folgt limn ∞  an/n=0

Nun, da hast du aber nicht richtig hingeguckt.

Thilo87 hat bei der von ihm genannten Folge an = ln(n) / n das Verhalten des Zählers mit dem des Nenners verglichen und festgestellt, dass der Zähler langsamer wächst, als der Nenner und daraus korrekterweise geschlossen, dass es sich bei seiner Folge um eine Nullfolge handeln muss.

Du hingegen betrachtest eine Folge ( bn=n / 2 ), bei der im Gegensatz zu der Folge von Thilo87 der Zähler schneller wächst als der Nenner, der konstant bleibt. Es ist leicht einzusehen, dass eine solche Folge bestimmt divergent ist. Mit der von Thilo87 genannten Folge hat deine Folge aber gar nichts zu tun, sie verhält sich völlig anders.

LieberJotEs,

hast du meinen ersten Post überhaupt gelesen?

Die zu beweisende Aussage ist gerade die, das der "Zähler langsamer wächst"

Die Folge n/2 wächst definitv nie schneller als die Folge n.

Was für eine Folge meinst du im zweitletzten Satz denn genau?

Und Thilo hat bei seiner Ungleichung die Folge ln(n) betrachtet, nicht ln(n)/n.

3 Antworten

+1 Daumen
Ich denke, dass man es so zeigen kann.

Allerdings würde ich es in diesem Falle anders machen:

Da sowohl f ( n ) = ln ( n ) als auch g ( n ) = n divergent sind, kann man die Regel von L'Hospital anwenden:
$$\lim _{ n\rightarrow \infty  }{ \frac { f(n) }{ g(n) }  } =\lim _{ n\rightarrow \infty  }{ \frac { f'(n) }{ g'(n) }  }$$
falls der Grenzwert auf der rechten Seite des Gleichheitszeichens existiert.

Also:
$$\lim _{ n\rightarrow \infty  }{ \frac { ln(n) }{ n }  } =\lim _{ n\rightarrow \infty  }{ \frac { \frac { 1 }{ n }  }{ 1 }  } =\lim _{ n\rightarrow \infty  }{ \frac { 1 }{ n }  } =0$$
Avatar von 32 k
Danke, die Regel von l'Hospital hatten wir in den Vorlesungen noch gar nicht, aber damit geht es natürlich. Für den allgemeinen Fall kann man es aber anscheinend nicht so machen (siehe den Kommentar von Anonym auf meine Frage).
Siehe dazu die Kommentare im Anschluss an die Fragestellung.
Siehe dazu den Kommentar zum Kommentar. Ferner habe ich nicht behauptet, dass es nicht so geht wie hier (ich habe ja soagr auf L'Hopital hingewiesen), nur, dass die dortige Argumentation von Thilo87 nicht schlüssig ist.
0 Daumen
Hi Thilo,

ich sehe da jetzt keinen Fehler, aber dennoch einiges an Umständlichkeit.

In einer Zeile (danke l'Hospital):

$$\lim_{n\to\infty} \frac{\ln(n)}{n} = l'H = \lim \frac{\frac1n}{1} = \lim\frac1n = 0$$

;)

Grüße
Avatar von 141 k 🚀
0 Daumen
Hallo Thilo87,

  ln(n) / n : Zähler und Nenner gehen gegen unendlich. Hier kann L´hopital angewendet werden.

  [ ln (n) ]´ / n´ = ( 1/ n ) / 1 = 1/ n . Da n gegen unendlich geht, geht der Ausdruck gegen 0.

  mfg Georg
Avatar von 122 k 🚀

Ein anderes Problem?

Stell deine Frage

Willkommen bei der Mathelounge! Stell deine Frage einfach und kostenlos

x
Made by a lovely community